Learning from Mistakes: Ratio Test Problem Solving

Click For Summary
The discussion focuses on understanding the ratio test in calculus, specifically a step in the solution involving factorial decomposition. The user initially misinterpreted the breakdown of (n-1)! into (n-1)(n-2)!. Clarification was provided that (n-1)! can indeed be expressed as (n-1)(n-2)!, which is a standard factorial decomposition. The conversation emphasizes the importance of grasping these mathematical concepts to avoid similar mistakes in the future. Overall, the exchange highlights a collaborative effort to enhance understanding of factorials in problem-solving.
mmont012
Messages
39
Reaction score
0

Homework Statement


I'm reviewing for a test and working on the practice problems for the ratio test that Pauls Online Notes gives. So here is given problem:
eq0001MP.gif


Here is his solution for the problem:
eq0002MP.gif
2. The attempt at a solution
I worked this out before I looked at the solution and I got it wrong. I had it right up until the 4th step. Can someone explain this step to me? I don't understand why the (n-1)! is broken up into (n-1)(n-2)! Isn't this supposed to be the decomposition of (n-2)!? I'm sorry if this is a stupid question, I'm just trying to learn from my mistakes so that I won't do this again.

Thanks for stopping by!
 
Physics news on Phys.org
Not sure I understand your question; (n-1)! =(n-1)(n-2)...2.1= (n-1)[(n-2)(n-3)...2.1]= (n-1)(n-2)! is a decomposition of (n-1)!
 
  • Like
Likes mmont012
Then what is the decomposition on (n-2)! ?
 
In this format, it is (n-2)(n-3)! . In general, (n-k)!= (n-k)(n-k-1)! if n>k, to avoid a -1!.
 
  • Like
Likes mmont012
Thank you so much! This helps me out tremendously!
 
Glad it helped, mmont012.
 
Question: A clock's minute hand has length 4 and its hour hand has length 3. What is the distance between the tips at the moment when it is increasing most rapidly?(Putnam Exam Question) Answer: Making assumption that both the hands moves at constant angular velocities, the answer is ## \sqrt{7} .## But don't you think this assumption is somewhat doubtful and wrong?

Similar threads

  • · Replies 4 ·
Replies
4
Views
1K
  • · Replies 2 ·
Replies
2
Views
2K
  • · Replies 7 ·
Replies
7
Views
2K
  • · Replies 2 ·
Replies
2
Views
2K
Replies
10
Views
1K
  • · Replies 6 ·
Replies
6
Views
2K
  • · Replies 2 ·
Replies
2
Views
1K
Replies
23
Views
2K
  • · Replies 7 ·
Replies
7
Views
2K
  • · Replies 6 ·
Replies
6
Views
2K